Absolutely continuous functions and sets of measure 0.

Click For Summary
If f: [a,b] -> R is absolutely continuous and E is a subset of [a,b] with measure zero, then the image f(E) also has measure zero. The definition of absolute continuity involves controlling the function's output based on the input intervals' lengths. To prove the statement, one can cover the set E with intervals and utilize the properties of absolute continuity to show that the measure of f(E) can be made arbitrarily small. The discussion emphasizes the need to connect the measure zero property of E with the behavior of f under absolute continuity. Ultimately, demonstrating that f(E) has measure zero follows from these principles.
glacier302
Messages
34
Reaction score
0

Homework Statement



Prove that if f: [a,b] -> R is absolutely continuous, and E ∁ [a,b] has measure zero, then f(E) has measure zero.

Homework Equations



A function f: [a,b] -> R is absolutely continuous if for every ε > 0 there is an δ > 0 such that for every finite sequence {(xj,xj')} of nonoverlapping intervals in [a,b] with ∑|xj'-xj| < δ, ∑|f(xj')-f(xj)| < ε .



The Attempt at a Solution



I think that there is an alternative definition of absolute continuity using countable intervals instead of finite intervals, and if I knew that the set E was countable I think I could go from there...but I don't know that E is countable; I only know that it has measure zero. So I'm not really sure where to start.

Any help would be much appreciated : )
 
Physics news on Phys.org
Well, if you can show that the measure of f(E) is less than any epsilon, you'll be done right? Fix an \epsilon &gt; 0. Furthermore, you know that E has zero measure, so can you cover it with intervals in a way that allows you to exploit absolute continuity?
 
Question: A clock's minute hand has length 4 and its hour hand has length 3. What is the distance between the tips at the moment when it is increasing most rapidly?(Putnam Exam Question) Answer: Making assumption that both the hands moves at constant angular velocities, the answer is ## \sqrt{7} .## But don't you think this assumption is somewhat doubtful and wrong?

Similar threads

Replies
7
Views
1K
Replies
1
Views
2K
Replies
4
Views
4K
  • · Replies 11 ·
Replies
11
Views
2K
Replies
30
Views
3K
  • · Replies 8 ·
Replies
8
Views
2K
  • · Replies 2 ·
Replies
2
Views
1K
  • · Replies 5 ·
Replies
5
Views
3K
Replies
26
Views
2K
Replies
1
Views
2K